Những câu hỏi liên quan
Lê Song Phương
Xem chi tiết
Mai Tiến Đỗ
Xem chi tiết
Minh Nguyễn Cao
Xem chi tiết
tth_new
31 tháng 7 2019 lúc 20:37

Em thử, sai thì thôi nha, chỗ đặt xong rồi thay vào P em ko biết mình có tính đúng hay sai nữa!

giả thiết \(\Leftrightarrow\frac{1}{x}+\frac{1}{y}+\frac{1}{z}=2\).

Đặt \(\left(\frac{1}{x};\frac{1}{y};\frac{1}{z}\right)\rightarrow\left(a;b;c\right)\) thì a + b + c = 2; a, b, c > 0 và:

\(P=\frac{a^2}{a+b}+\frac{b^2}{b+c}+\frac{c^2}{c+a}\)

\(\ge\frac{\left(a+b+c\right)^2}{2\left(a+b+c\right)}=\frac{a+b+c}{2}=\frac{2}{2}=1\)

Đẳng thức xảy ra khi a = b = c = 2/3 hay \(x=y=z=\frac{3}{2}\)

Nguyễn Văn Vũ
Xem chi tiết
Nguyễn Văn Vũ
Xem chi tiết
Trần Hữu Ngọc Minh
Xem chi tiết
_ɦყυ_
16 tháng 11 2017 lúc 23:09

ta có:2(y+z)=x(yz-1)

=>2y+2z=xyz-x

=>2y+2z+x=xyz

mik ko làm tiếp đc do thiếu đ/k

Quang Đẹp Trai
Xem chi tiết
Akai Haruma
10 tháng 8 2023 lúc 23:23

Lời giải:
Áp dụng BĐT Bunhiacopxky:

$\text{VT}(1^2+1^2+1^2)\geq (1+\frac{x}{y+z}+1+\frac{y}{x+z}+1+\frac{z}{x+y})^2$

$\Leftrightarrow 3\text{VT}\geq (3+\frac{x}{y+z}+\frac{y}{x+z}+\frac{z}{x+y})^2$

$ = \left[3+\frac{x^2}{xy+xz}+\frac{y^2}{yz+yx}+\frac{z^2}{zy+zx}\right]^2$

$\geq \left[3+\frac{(x+y+z)^2}{2(xy+yz+xz)}\right]^2$

$\geq \left[3+\frac{3(xy+yz+xz)}{2(xy+yz+xz)}\right]^2=\frac{81}{4}$

$\Rightarrow \text{VT}\geq \frac{27}{4}$

Dấu "=" xảy ra khi $x=y=z>0$

Vũ Tuệ Lâm
10 tháng 8 2023 lúc 23:38

Áp dụng BĐT Bunhiacopxky:

VT(12+12+12)≥(1+��+�+1+��+�+1+��+�)2

⇔3VT≥(3+��+�+��+�+��+�)2

=[3+�2��+��+�2��+��+�2��+��]2

≥[3+(�+�+�)22(��+��+��)]2

≥[3+3(��+��+��)2(��+��+��)]2=814

⇒VT≥274

Dấu "=" xảy ra khi �=�=�>0

Bùi Khắc Tuấn Khải
Xem chi tiết
1st_Parkour
2 tháng 6 2016 lúc 9:26

mk ko bit

Nhớ Mãi Mái Trường Xưa
3 tháng 6 2016 lúc 16:56

mik tính ko ra

Lionel Messi
Xem chi tiết
Cù Đức Anh
4 tháng 12 2021 lúc 22:33

sai đề

Nguyễn Việt Lâm
4 tháng 12 2021 lúc 23:04

Theo nguyên lý Dirichlet, trong 3 số x;y;z luôn có 2 số cùng phía so với \(\dfrac{1}{2}\)

Không mất tính tổng quát, giả sử đó là y và z 

\(\Rightarrow\left(y-\dfrac{1}{2}\right)\left(z-\dfrac{1}{2}\right)\ge0\Leftrightarrow yz-\dfrac{1}{2}\left(y+z\right)+\dfrac{1}{4}\ge0\)

\(\Leftrightarrow y+z-yz\le\dfrac{1}{2}+yz\)

Mặt khác từ giả thiết:

\(1-x^2=y^2+z^2+2xyz\ge2yz+2xyz\)

\(\Leftrightarrow\left(1-x\right)\left(1+x\right)\ge2yz\left(1+x\right)\)

\(\Leftrightarrow1-x\ge2yz\)

\(\Rightarrow yz\le\dfrac{1-x}{2}\)

Do đó:

\(A=yz+x\left(y+z-yz\right)\le yz+x\left(\dfrac{1}{2}+yz\right)=\dfrac{1}{2}x+yz\left(x+1\right)\le\dfrac{1}{2}x+\left(\dfrac{1-x}{2}\right)\left(x+1\right)\)

\(\Rightarrow A\le-\dfrac{1}{2}x^2+\dfrac{1}{2}x+\dfrac{1}{2}=-\dfrac{1}{2}\left(x-\dfrac{1}{2}\right)^2+\dfrac{5}{8}\le\dfrac{5}{8}\)

\(A_{max}=\dfrac{5}{8}\) khi \(\left(x;y;z\right)=\left(\dfrac{1}{2};\dfrac{1}{2};\dfrac{1}{2}\right)\)